
PrepTest 90 Game 3: In Out Game Re-Using Elements // Logic Games [#30] [LSAT Analytical Reasoning]
polymathematic
Views: 1226
Like: 33
This has been an interesting section of games (LSAT PrepTest 90). Each game so far has had a mostly basic setup, but with a wrinkle of some kind thrown in. For today’s game, the third game from PrepTest 90, it’s a basic in and out set-up with the twist that one of the elements (but only one!) gets used twice: once on the in-side (in this case “Fall”) and one on the our-side (“Spring”).
Subscribe: | Enable ALL push notifications 🔔
This twist affects both of the conditional clues, and requires that we symbolize them a little differently than we would in a typical in-and-out setup.
I continue to work through all four-hundred-ish official LSAC releases. As always, if there’s a particular game you’d like to see, comment down below, and I’ll move it up in the order.
Watch more LSAT videos:
Follow Tim Ricchuiti:
TikTok:
Twitter:
Instagram:
Reddit:
lsat flex
01.01.2023
Love to get tutoring. I just emailed you to see if you have time.
This game was hard and time consuming.
Please explain the 93 pt logic games section, if you can. It would be of great help!
I drew out six game boards based off the MV rule and one board didn't work because it contradicted the first two rules. Is this correct? I couldn't do it without the boards.
You sir are amazing, thank you for uploading these videos.
This was fun!
Please slow down. You speeding through does not help in figuring out the strategy.
Thank you soooo much for this! I couldn't figure out why doing the contrapositives the normal way didn't work (I knew they didn't when I went to do Q16 and no answer choices worked). Thank you so much for explaining how we should have approached the contrapositives in the beginning of the video. Are there any other games like this that you know of? (In/out with repeating elements)
My question is: for #17, how is E not also correct? Either V and or M must be in the Spring and Fall. So if attempting to prove E wrong, V would have to be in the fall making it HMV for fall. Then it would have to be TMO for Spring but if M is in spring then H has to be too… Don't V and T both have to be in the Spring?